Solve Disk Rotation Problem: Conservation of Energy vs Momentum

In summary, the question asks for the new angular velocity ω when a disk of mass M and radius R rotates at angular velocity ω0 and another disk of the same mass and radius r is dropped on top of it, assuming a negligible loss of energy to friction. The correct answer is ω=((R**2)ω0)/(R**2+r**2) and can be obtained by setting initial and final angular momentum equal. However, the conversation also discusses solving the problem using conservation of energy, which leads to a different answer due to the inelastic collision between the two disks. The person asking the question realizes that the problem in the book contains an error, as the sentence "assuming a negligible loss of energy to friction"
  • #1
Teachme
72
0
Ok here is the question that is being asked. Note that I know how to solve it one way, but when I go about trying to solve this problem another way that I think should give the the same result, I don't end up with the same result.

Question:
A disk of mass M and radius R rotates at angular velocity ω0. Another disk of mass M and radius r is dropped on top of the rotating disk, in its center, causing both disks to spin at a new angular velocity ω. Assuming a negligible loss of energy to friction what is ω?

Ok so this problem is pretty easy just set initial angular momentum to final angular momentum

I(large disk)(ω0)=I(large disk)(ω)+I(small disk)(ω) and just solve for ω.

which gives me an answer of ω=((R**2)ω0)/(R**2+r**2)

So I know that is the correct answer. But then I also thought that I should be able to solve this using conservation of energy. I set this up like so

(1/2)I(large disk)(ω0**2)=(1/2)I(large disk)(ω**2)+(1/2)I(small disk)(w**2)

however when I solve I get

ω=R(ω0)/sqrt(R**2+r**2)

I checked my work multiple times and cannot find an error. I just don't know where my logic is going wrong. I mean if there is conservation of angular momentum doesn't that imply conservation of energy? Or since you are adding mass to the system can I not set up my energy conservation equation that way because I am not accounting for the rest energy of the second disk? I guess it is an inelastic collision when you drop the second disk on the top of the first so maybe that is why as well. But they say assuming a negligible loss to friction which throws me off. I mean isn't there significant loss of friction in inelastic collisions or do they just mean a small loss compared to the total rotational energy.

Thanks for reading
 
Last edited:
Physics news on Phys.org
  • #2
Teachme said:
Question:
A disk of mass M and radius R rotates at angular velocity ω0. Another disk of mass M and radius r is dropped on top of the rotating disk, in its center, causing both disks to spin at a new angular velocity ω. Assuming a negligible loss of energy to friction what is ω?
That last sentence is problematic. The only way for both disks to end up rotating together is for friction to act, and that means kinetic energy is not conserved. It's an inelastic collision.

Was that sentence really part of the problem?

I checked my work multiple times and cannot find an error. I just don't know where my logic is going wrong. I mean if there is conservation of angular momentum doesn't that imply conservation of energy?
No, not at all!
 
  • #3
Yeah that's exactly what I was just thinking. Yeah it is a problem in a conquering the physics GRE book by Yoni Kahn and Adam Anderson. Well I guess not word for word. The last sentence is actually. "Assuming negligible a negligible loss of energy to friction, what is w?" I figured the first negligible was a typo.
 
  • #4
Teachme said:
Yeah it is a problem in a conquering the physics GRE book by Yoni Kahn and Adam Anderson. Word for word.
Yikes. They messed up!
 
  • #6
Wait sorry I read wrong. It says "Assuming negligible a negligible loss of energy to friction, what is w?" I figured the first negligible was a typo but maybe my english reading is wrong and that changes the sentence.
 
  • #7
AHH THANK you sooo much. You just solved another problem I had with the book as well that I thought was weird! Just solved two problems that I was stuck on. Saved me from posting for the other. Thanks so much!
 

1. What is the disk rotation problem and why is it important in science?

The disk rotation problem refers to the conservation of energy and momentum in a rotating disk. It is important in science because it helps us understand the principles of energy and momentum conservation, which are fundamental concepts in physics.

2. What is the difference between conservation of energy and conservation of momentum?

The conservation of energy states that the total amount of energy in a closed system remains constant over time, whereas the conservation of momentum states that the total momentum of a closed system remains constant over time. In the disk rotation problem, both energy and momentum are conserved.

3. How does the conservation of energy and momentum apply to the disk rotation problem?

In the disk rotation problem, the initial energy and momentum of the disk are conserved as it rotates. This means that the total energy and momentum of the disk remains constant, even as the rotational speed changes.

4. Can the conservation of energy and momentum be used to solve the disk rotation problem?

Yes, the conservation of energy and momentum can be used to solve the disk rotation problem. By setting up and solving equations that describe the energy and momentum of the system, we can determine the final rotational speed of the disk.

5. What are some real-life applications of the disk rotation problem?

The disk rotation problem has many real-life applications, such as in the design of rotating machinery, like turbines and engines. It is also important in understanding the orbits of planets and other celestial bodies, as well as the rotation of galaxies.

Similar threads

Replies
2
Views
855
Replies
12
Views
3K
Replies
1
Views
1K
  • Classical Physics
Replies
5
Views
1K
  • Mechanics
Replies
12
Views
1K
Replies
19
Views
1K
Replies
30
Views
1K
Replies
1
Views
435
  • Introductory Physics Homework Help
Replies
8
Views
1K
Back
Top